LSAT and Law School Admissions Forum

Get expert LSAT preparation and law school admissions advice from PowerScore Test Preparation.

User avatar
 Dave Killoran
PowerScore Staff
  • PowerScore Staff
  • Posts: 5852
  • Joined: Mar 25, 2011
|
#71218
This game is discussed in our Podcast: LSAT Podcast Episode 31: The September 2019 LSAT Logic Games Section


Complete Question Explanation
(The complete setup for this game can be found here: https://forum.powerscore.com/lsat/viewtopic.php?t=31306)

The correct answer choice is (B).


Answer choice (A): This answer violates the second rule (M is 3 but N is 6), and is therefore incorrect.

Answer choice (B): This is the correct answer choice. It also conveniently mirrors the hypothetical we produced in the setup to show how the J/P/S not-block separation worked, although that solution has no impact on you being able to identify this as the correct answer.

Answer choice (C): This answer violates the first rule (G is 7), and is therefore incorrect.

Answer choice (D): This answer violates the third rule (M is 3 but H is 7), and is therefore incorrect.

Answer choice (E): This answer violates the last rule (S is 1 and J is 7, but 1 and 7 are next to each other—note how they waited until the end of the answers to test this!), and is therefore incorrect.
 mcmil1ca
  • Posts: 1
  • Joined: Jan 12, 2020
|
#73234
For this one I got B, using process of elimination.
 Adam Tyson
PowerScore Staff
  • PowerScore Staff
  • Posts: 5153
  • Joined: Apr 14, 2011
|
#73237
Exactly right, mcmil1ca, and elimination (what we call the Rule-By-Rule method) is the best way to get there! Run through each of the rules and cross out any answer that breaks that rule. So,,,'

H or J is last - that knocks out answer C

N is before M - that knocks out answer A

H and M must be adjacent - D is out

P and S are not adjacent - both of the remaining answers pass that test

J is not next to P or S - answer E violates that (because the first thing is next to the last thing)

Answer B stood the test and is the correct answer! Rule by rule for the win! Nice job.

Get the most out of your LSAT Prep Plus subscription.

Analyze and track your performance with our Testing and Analytics Package.